Free GMAT Prep Hour: Data Sufficiency Traps & Patterns

Поділитися
Вставка
  • Опубліковано 16 вер 2024

КОМЕНТАРІ • 26

  • @Housemusiclyfe
    @Housemusiclyfe 2 роки тому +5

    This is really helpful and well explained! Thanks Ceilidh

  • @josephzenteno8293
    @josephzenteno8293 5 років тому +2

    She seems great took 9 week course with another instructor but she seems awesome

  • @divyanshutyagi5080
    @divyanshutyagi5080 2 роки тому

    In ceilidh question.... question dont mention that the price has to be ascending order.

  • @angie_vr3
    @angie_vr3 2 роки тому

    about the books problem. why non-integer numbers were ruled out if the problem does not indicate that the cost of each book is an INT? thank you!!!

    • @manhattanprepgmat6791
      @manhattanprepgmat6791  2 роки тому +1

      Thanks for your question! You are correct that, on the real GMAT, the problem would either specify that they were integers or they wouldn't have to be integers. The test will sometimes make you assume that something is an integer, but only on things that truly can't be split up (eg, people, flowers, marbles you're picking out of a bowl).

    • @angie_vr3
      @angie_vr3 2 роки тому

      @@manhattanprepgmat6791 thanks for the explanation! :)

  • @Forever.Grateful
    @Forever.Grateful 3 роки тому +1

    Question 1: as mentioned, both statements are always true , then why didn't we consider 13 dollars for poetry book to be true?

    • @filiphellner6414
      @filiphellner6414 3 роки тому +2

      Well, since it was true the 15 dollar would be the median. BUT, even if we didn't have that information the median would still be 15 dollars right? because we can not have three books where one costs 15 dollars and sum is 45, without the 15 being median. We would either need two books that costs less than 15 dollars for it to be the highest and then it does not add up to 45. And if 15 is going to be the lowest we need two books priced above 15 dollars and that is not possible either if the sum is 45. Meaning we know that the median is 15 without even knowing statement a.

    • @safejourney6583
      @safejourney6583 10 місяців тому

      What if the price of the third book is 17$ so the sum of the three books will be 13+15+17=45 and on this case the average will be 15 and the median will be 15 also . I still thinking the write answer is C not B

  • @averygobbo9286
    @averygobbo9286 3 роки тому

    Is there a link for the answer/proof of the last question somewhere?

    • @manhattanprepgmat6791
      @manhattanprepgmat6791  3 роки тому +7

      This is a really tough one! It's based on an official problem that you can find in the free problem set on mba.com. The question stem is describing p as a factor of n-basically, is there some factor of n that is greater than 1 but less than n itself? If so, then this number can't be a prime number, so that's how Ceilidh translated the question itself into "is n not a prime number?" Statement (1) tells you that n is greater than some really big number, but it could be anything greater than that; some of those numbers will be prime and some won't be, so that's not sufficient. Statement (2) is the crazy-hard one. There's a math rule that states this: If you have two numbers that are both divisible by the same integer, and you add them together, the sum will still be divisible by that same integer. For example, if you add 35 and 42, both of which are divisible by 7, then the sum will also be divisible by 7. This rule applies to every single value in the range of really big numbers given in statement (2). For example, the first number in the range is 19! + 3. The value for 19! is found by multiplying 1*2*3*4*...*19. That value is divisible by 3 (because 3 is one of the numbers you have to multiply to create it)...and then you add 3 to it, so the resulting sum is also divisible by 3. The next value in the list is 19! + 4 and, ditto, both numbers are divisible by 4 so their sum is divisible by 4. You can do this all the way up through 19! + 19, so every number in that range does have a factor that is greater than 1 but less than n. In other words, every number in that range is *not* a prime number.

    • @averygobbo9286
      @averygobbo9286 3 роки тому +1

      @@manhattanprepgmat6791 Thank you very much, super helpful and clear explanation!

    • @nemonymous89
      @nemonymous89 2 роки тому

      That seems like a college-level math rule related to number theory?!?

  • @healthyummytummy
    @healthyummytummy 3 роки тому

    Review log link please!

    • @manhattanprepgmat6791
      @manhattanprepgmat6791  3 роки тому

      Here's a blog post about making an error log for yourself! www.manhattanprep.com/gmat/blog/error-log-the-1-way-to-raise-your-gmat-score/

  • @cekirdekci32
    @cekirdekci32 3 роки тому

    questions are pretty clear. I dont understand why overthingking or over explaining. i ll simply never understand.

  • @likquidsteel
    @likquidsteel 3 роки тому +9

    @52:00 - that is false. The answer is C. Statement 1 tells us that each trick or treater received 12 pieces of candy (3+4+5). Statement 2 tells us the total number of candies distributed: 96 (24+32+40). 96/12 = 8. That's eight trick-or-treaters.

    • @Bloo_M
      @Bloo_M 3 роки тому +4

      But statement one does NOT tell us how many pieces each trick or treater received. It is just the ratio, nothing more. If there was just 1 or 2 or 4 trick or treaters, the sentences 1 and 2 would still be true.

    • @Heroesoftomorrow
      @Heroesoftomorrow 3 роки тому

      @@Bloo_M Statement one tells us the total number of candies each trick or treater gets. it is stated clearly there so why ignore it? The number of chocolates, gummy bears and lollipops that EACH trick or treater received was in the ration 3:4:5 meaning in total each kid gets 12 candies out of the total 96 candies that was shared per the summation of all the various types of candies there are. and if each child had the same number so 12 candies per child. Dividing the 96 by 12 gives us the total number of trick or treaters who came to the door giving us 8! Also the opening line of the STEM mentioned TRICK OR TREATERS, indicating more than one trick or treater came to the her door.

    • @Bloo_M
      @Bloo_M 3 роки тому +1

      @@Heroesoftomorrow Neither the question nor statement 1 give us the information, that there are 96 candy bars. That information is given to us by statement 2. But we have to ignore statement 2, to answer the question, if statement 1 alone is sufficient.

    • @Heroesoftomorrow
      @Heroesoftomorrow 3 роки тому

      @@Bloo_M yes we do need to ignore statement 2 to answer statement 1, but do you agree it is clearly more than 1 child from the stem and that each child gets 12 candies in total from the first statement?

    • @Bloo_M
      @Bloo_M 3 роки тому

      @@Heroesoftomorrow could be 12, could be 24, could be 36 etc. anything that's divisible by 12 (3+4+5). so we cannot know with only the information of the ratio.

  • @maxponce4808
    @maxponce4808 3 роки тому

    c